1994 AHSME Problems/Problem 10

Revision as of 21:26, 27 June 2014 by TheMaskedMagician (talk | contribs) (Created page with "==Problem== For distinct real numbers <math>x</math> and <math>y</math>, let <math>M(x,y)</math> be the larger of <math>x</math> and <math>y</math> and let <math>m(x,y)</math> be...")
(diff) ← Older revision | Latest revision (diff) | Newer revision → (diff)

Problem

For distinct real numbers $x$ and $y$, let $M(x,y)$ be the larger of $x$ and $y$ and let $m(x,y)$ be the smaller of $x$ and $y$. If $a<b<c<d<e$, then \[M(M(a,m(b,c)),m(d,m(a,e)))=\] $\textbf{(A)}\ a \qquad\textbf{(B)}\ b \qquad\textbf{(C)}\ c \qquad\textbf{(D)}\ d \qquad\textbf{(E)}\ e$

Solution